Solving a Double Integral: Where is the Error?

Click For Summary
The discussion centers on solving a double integral, specifically the integral of y^2 over the region defined by -1 ≤ y ≤ 1 and -y-2 ≤ x ≤ y. The user initially set up the integral correctly but mistakenly reversed the bounds when integrating, leading to an incorrect result of 0 instead of the expected 4/3. Participants pointed out the error in the bounds and clarified that the integral should be evaluated with the correct limits. The user acknowledged the mistake and corrected the expression for the integrand. The conversation emphasizes the importance of proper bounds in double integrals to achieve accurate results.
tnutty
Messages
324
Reaction score
1

Homework Statement



\int_{D}\int y^2
where D = {(x,y) | -1 \leq y \leq1, -y-2\leq x\leq y

The integral I set up is below :

\int^{1}_{-1} \int^{y}_{-y-2} y^2 dx dy

From that I get the answer 0, but the book says its 4/3.

I get 0 because It reduces to this integral :

\int^{-1}_{1} 2y^3 + 2y

Any idea where I could be wrong?
 
Physics news on Phys.org
tnutty said:

Homework Statement



\int_{D}\int y^2
where D = {(x,y) | -1 \leq y \leq1, -y-2\leq x\leq y

The integral I set up is below :

\int^{1}_{-1} \int^{y}_{-y-2} y^2 dx dy

From that I get the answer 0, but the book says its 4/3.

I get 0 because It reduces to this integral :

\int^{-1}_{1} 2y^3 + 2y

Any idea where I could be wrong?

In your last step you reversed your bounds, it should be (-1,1) not (1,-1) as you wrote. Also the final step should be \int2y3 +2y2 dy with the bounds (-1 to 1).
 
Last edited:
Hi tnutty! :smile:
tnutty said:
Any idea where I could be wrong?

erm :redface:

2y3 + 2y2 ? :wink:
 
I mean to write 2y^3 + 2y^2. But its was the bounds. Thanks guys.
 
Question: A clock's minute hand has length 4 and its hour hand has length 3. What is the distance between the tips at the moment when it is increasing most rapidly?(Putnam Exam Question) Answer: Making assumption that both the hands moves at constant angular velocities, the answer is ## \sqrt{7} .## But don't you think this assumption is somewhat doubtful and wrong?

Similar threads

  • · Replies 19 ·
Replies
19
Views
2K
  • · Replies 1 ·
Replies
1
Views
2K
  • · Replies 105 ·
4
Replies
105
Views
6K
  • · Replies 4 ·
Replies
4
Views
2K
Replies
2
Views
2K
Replies
4
Views
2K
Replies
7
Views
1K
Replies
20
Views
2K
  • · Replies 10 ·
Replies
10
Views
2K
Replies
3
Views
2K